Prove that $sqrt[3]p+sqrt[3]p^5$ is irrational if $p$ is prime [on hold]How to prove that $sqrt 3$ is an irrational number?Prove $x = sqrt[100]sqrt3 + sqrt2 + sqrt[100]sqrt3 - sqrt2$ is irrationalProve that $sqrt 2 +sqrt 3$ is irrational.Show that $sqrt[3]2 + sqrt[3]4$ is irrationalProve that $sqrt[5]672$ is irrationalProve that $sqrt3+ sqrt5+ sqrt7$ is irrationalProving $sqrt[4]4$ is irrationalLet $p,q$ be irrational numbers, such that, $p^2$ and $q^2$ are relatively prime. Show that $sqrtpq$ is also irrational.Square root of a prime is irrationalProving $sqrt3 + sqrt[3]2$ to be irrational

Why does cron require MTA for logging?

What's the 'present simple' form of the word "нашла́" in 3rd person singular female?

What ability score modifier does a javelin's damage use?

How can I find out information about a service?

Which classes are needed to have access to every spell in the PHB?

Shifting between bemols and diesis in the key signature

I reported the illegal activity of my boss to his boss. My boss found out. Now I am being punished. What should I do?

Doesn't allowing a user mode program to access kernel space memory and execute the IN and OUT instructions defeat the purpose of having CPU modes?

How to resolve: Reviewer #1 says remove section X vs. Reviewer #2 says expand section X

What do you call someone who likes to pick fights?

How many characters using PHB rules does it take to be able to have access to any PHB spell at the start of an adventuring day?

Outlet with 3 sets of wires

Power Strip for Europe

PTIJ: Why does only a Shor Tam ask at the Seder, and not a Shor Mu'ad?

Why is a very small peak with larger m/z not considered to be the molecular ion?

When Schnorr signatures are part of Bitcoin will it be possible validate each block with only one signature validation?

Professor forcing me to attend a conference, I can't afford even with 50% funding

MySQL importing CSV files really slow

Why is gluten-free baking possible?

Signed and unsigned numbers

Was it really inappropriate to write a pull request for the company I interviewed with?

Would an aboleth's Phantasmal Force lair action be affected by Counterspell, Dispel Magic, and/or Slow?

Specifying a starting column with colortbl package and xcolor

What is the generally accepted pronunciation of “topoi”?



Prove that $sqrt[3]p+sqrt[3]p^5$ is irrational if $p$ is prime [on hold]


How to prove that $sqrt 3$ is an irrational number?Prove $x = sqrt[100]sqrt3 + sqrt2 + sqrt[100]sqrt3 - sqrt2$ is irrationalProve that $sqrt 2 +sqrt 3$ is irrational.Show that $sqrt[3]2 + sqrt[3]4$ is irrationalProve that $sqrt[5]672$ is irrationalProve that $sqrt3+ sqrt5+ sqrt7$ is irrationalProving $sqrt[4]4$ is irrationalLet $p,q$ be irrational numbers, such that, $p^2$ and $q^2$ are relatively prime. Show that $sqrtpq$ is also irrational.Square root of a prime is irrationalProving $sqrt3 + sqrt[3]2$ to be irrational













-1












$begingroup$


Prove that $sqrt[3]p+sqrt[3]p^5$ is irrational when $p$ is a prime.



First I suppose $x=sqrt[3]p+sqrt[3]p^5$. Cubing gives
$$x^3=p+p^5+p^2x$$
And then what properties of prime, and how to test its irrationallity?










share|cite|improve this question











$endgroup$



put on hold as off-topic by uniquesolution, Cesareo, Carl Mummert, B. Goddard, Parcly Taxel 23 hours ago


This question appears to be off-topic. The users who voted to close gave this specific reason:


  • "This question is missing context or other details: Please provide additional context, which ideally explains why the question is relevant to you and our community. Some forms of context include: background and motivation, relevant definitions, source, possible strategies, your current progress, why the question is interesting or important, etc." – uniquesolution, Cesareo, Carl Mummert, B. Goddard, Parcly Taxel
If this question can be reworded to fit the rules in the help center, please edit the question.















  • $begingroup$
    Are you sure that is irrational?
    $endgroup$
    – uniquesolution
    yesterday















-1












$begingroup$


Prove that $sqrt[3]p+sqrt[3]p^5$ is irrational when $p$ is a prime.



First I suppose $x=sqrt[3]p+sqrt[3]p^5$. Cubing gives
$$x^3=p+p^5+p^2x$$
And then what properties of prime, and how to test its irrationallity?










share|cite|improve this question











$endgroup$



put on hold as off-topic by uniquesolution, Cesareo, Carl Mummert, B. Goddard, Parcly Taxel 23 hours ago


This question appears to be off-topic. The users who voted to close gave this specific reason:


  • "This question is missing context or other details: Please provide additional context, which ideally explains why the question is relevant to you and our community. Some forms of context include: background and motivation, relevant definitions, source, possible strategies, your current progress, why the question is interesting or important, etc." – uniquesolution, Cesareo, Carl Mummert, B. Goddard, Parcly Taxel
If this question can be reworded to fit the rules in the help center, please edit the question.















  • $begingroup$
    Are you sure that is irrational?
    $endgroup$
    – uniquesolution
    yesterday













-1












-1








-1





$begingroup$


Prove that $sqrt[3]p+sqrt[3]p^5$ is irrational when $p$ is a prime.



First I suppose $x=sqrt[3]p+sqrt[3]p^5$. Cubing gives
$$x^3=p+p^5+p^2x$$
And then what properties of prime, and how to test its irrationallity?










share|cite|improve this question











$endgroup$




Prove that $sqrt[3]p+sqrt[3]p^5$ is irrational when $p$ is a prime.



First I suppose $x=sqrt[3]p+sqrt[3]p^5$. Cubing gives
$$x^3=p+p^5+p^2x$$
And then what properties of prime, and how to test its irrationallity?







number-theory irrational-numbers rationality-testing






share|cite|improve this question















share|cite|improve this question













share|cite|improve this question




share|cite|improve this question








edited yesterday









Parcly Taxel

43.5k1375104




43.5k1375104










asked yesterday









HeartHeart

29318




29318




put on hold as off-topic by uniquesolution, Cesareo, Carl Mummert, B. Goddard, Parcly Taxel 23 hours ago


This question appears to be off-topic. The users who voted to close gave this specific reason:


  • "This question is missing context or other details: Please provide additional context, which ideally explains why the question is relevant to you and our community. Some forms of context include: background and motivation, relevant definitions, source, possible strategies, your current progress, why the question is interesting or important, etc." – uniquesolution, Cesareo, Carl Mummert, B. Goddard, Parcly Taxel
If this question can be reworded to fit the rules in the help center, please edit the question.







put on hold as off-topic by uniquesolution, Cesareo, Carl Mummert, B. Goddard, Parcly Taxel 23 hours ago


This question appears to be off-topic. The users who voted to close gave this specific reason:


  • "This question is missing context or other details: Please provide additional context, which ideally explains why the question is relevant to you and our community. Some forms of context include: background and motivation, relevant definitions, source, possible strategies, your current progress, why the question is interesting or important, etc." – uniquesolution, Cesareo, Carl Mummert, B. Goddard, Parcly Taxel
If this question can be reworded to fit the rules in the help center, please edit the question.











  • $begingroup$
    Are you sure that is irrational?
    $endgroup$
    – uniquesolution
    yesterday
















  • $begingroup$
    Are you sure that is irrational?
    $endgroup$
    – uniquesolution
    yesterday















$begingroup$
Are you sure that is irrational?
$endgroup$
– uniquesolution
yesterday




$begingroup$
Are you sure that is irrational?
$endgroup$
– uniquesolution
yesterday










1 Answer
1






active

oldest

votes


















2












$begingroup$

By Eisenstein's criterion, $X^3-p$ is irreducible over $Bbb Q$. Therefore $1$, $sqrt[3]p$ and $sqrt[3]p^2$ are linearly independent over $Bbb Q$.



Now observe that $x=sqrt[3]p+psqrt[3]p^2$.






share|cite|improve this answer









$endgroup$












  • $begingroup$
    At this level one should really justify the inference "Therefore...."
    $endgroup$
    – Bill Dubuque
    yesterday


















1 Answer
1






active

oldest

votes








1 Answer
1






active

oldest

votes









active

oldest

votes






active

oldest

votes









2












$begingroup$

By Eisenstein's criterion, $X^3-p$ is irreducible over $Bbb Q$. Therefore $1$, $sqrt[3]p$ and $sqrt[3]p^2$ are linearly independent over $Bbb Q$.



Now observe that $x=sqrt[3]p+psqrt[3]p^2$.






share|cite|improve this answer









$endgroup$












  • $begingroup$
    At this level one should really justify the inference "Therefore...."
    $endgroup$
    – Bill Dubuque
    yesterday
















2












$begingroup$

By Eisenstein's criterion, $X^3-p$ is irreducible over $Bbb Q$. Therefore $1$, $sqrt[3]p$ and $sqrt[3]p^2$ are linearly independent over $Bbb Q$.



Now observe that $x=sqrt[3]p+psqrt[3]p^2$.






share|cite|improve this answer









$endgroup$












  • $begingroup$
    At this level one should really justify the inference "Therefore...."
    $endgroup$
    – Bill Dubuque
    yesterday














2












2








2





$begingroup$

By Eisenstein's criterion, $X^3-p$ is irreducible over $Bbb Q$. Therefore $1$, $sqrt[3]p$ and $sqrt[3]p^2$ are linearly independent over $Bbb Q$.



Now observe that $x=sqrt[3]p+psqrt[3]p^2$.






share|cite|improve this answer









$endgroup$



By Eisenstein's criterion, $X^3-p$ is irreducible over $Bbb Q$. Therefore $1$, $sqrt[3]p$ and $sqrt[3]p^2$ are linearly independent over $Bbb Q$.



Now observe that $x=sqrt[3]p+psqrt[3]p^2$.







share|cite|improve this answer












share|cite|improve this answer



share|cite|improve this answer










answered yesterday









Lord Shark the UnknownLord Shark the Unknown

106k1161133




106k1161133











  • $begingroup$
    At this level one should really justify the inference "Therefore...."
    $endgroup$
    – Bill Dubuque
    yesterday

















  • $begingroup$
    At this level one should really justify the inference "Therefore...."
    $endgroup$
    – Bill Dubuque
    yesterday
















$begingroup$
At this level one should really justify the inference "Therefore...."
$endgroup$
– Bill Dubuque
yesterday





$begingroup$
At this level one should really justify the inference "Therefore...."
$endgroup$
– Bill Dubuque
yesterday




Popular posts from this blog

Lowndes Grove History Architecture References Navigation menu32°48′6″N 79°57′58″W / 32.80167°N 79.96611°W / 32.80167; -79.9661132°48′6″N 79°57′58″W / 32.80167°N 79.96611°W / 32.80167; -79.9661178002500"National Register Information System"Historic houses of South Carolina"Lowndes Grove""+32° 48' 6.00", −79° 57' 58.00""Lowndes Grove, Charleston County (260 St. Margaret St., Charleston)""Lowndes Grove"The Charleston ExpositionIt Happened in South Carolina"Lowndes Grove (House), Saint Margaret Street & Sixth Avenue, Charleston, Charleston County, SC(Photographs)"Plantations of the Carolina Low Countrye

random experiment with two different functions on unit interval Announcing the arrival of Valued Associate #679: Cesar Manara Planned maintenance scheduled April 23, 2019 at 00:00UTC (8:00pm US/Eastern)Random variable and probability space notionsRandom Walk with EdgesFinding functions where the increase over a random interval is Poisson distributedNumber of days until dayCan an observed event in fact be of zero probability?Unit random processmodels of coins and uniform distributionHow to get the number of successes given $n$ trials , probability $P$ and a random variable $X$Absorbing Markov chain in a computer. Is “almost every” turned into always convergence in computer executions?Stopped random walk is not uniformly integrable

How should I support this large drywall patch? Planned maintenance scheduled April 23, 2019 at 00:00UTC (8:00pm US/Eastern) Announcing the arrival of Valued Associate #679: Cesar Manara Unicorn Meta Zoo #1: Why another podcast?How do I cover large gaps in drywall?How do I keep drywall around a patch from crumbling?Can I glue a second layer of drywall?How to patch long strip on drywall?Large drywall patch: how to avoid bulging seams?Drywall Mesh Patch vs. Bulge? To remove or not to remove?How to fix this drywall job?Prep drywall before backsplashWhat's the best way to fix this horrible drywall patch job?Drywall patching using 3M Patch Plus Primer